You are on page 1of 40

SECTION-A : ENGLISH LANGUAGE

Directions (Q.1-Q.30): Read the passages carefully and answer the questions.

Passage (Q.1-Q.5): On Monday, Assam and Mizoram clashed over a disputed boundary. Five policemen and one
civilian (all from Assam) were killed and nearly 50 others were injured. The bloodiest-ever incident between the
two states was the culmination of the discontent that had been simmering since October 2020 over incidents of
violence and Assam9s residents blocking NH-306, the lifeline to Mizoram, for 12 days. While Assam claims
Mizoram has encroached on its boundary, Mizoram maintains the area belongs to the state.
Mizoram has accused Assam police personnel of entering its territory, indulging in violence, and claimed that it
only retaliated after tear gas canisters and grenades were hurled by Assam9s security forces. Assam has blamed
Mizoram of breaching existing agreements and status quo on the border, and accused the Mizoram police of firing
on its security forces and civilians using light machine guns. A violent clash of this nature between two Indian
states represents a constitutional breakdown. And the fact that the clash played out on Twitter, with chief ministers
(CMs) of both states sparring and urging home minister Amit Shah to intervene, represents a political and
institutional failure. Incidentally, Mr Shah had met the CMs of all Northeastern states over the weekend in Shillong
where long pending inter-state border disputes were discussed.
Mizoram9s border dispute with Assam goes back to 1972 when the former was first carved out as a Union Territory
(it became a state in 1987). Assam also has border disputes with Meghalaya, Nagaland and Arunachal Pradesh.
With states in the region either ruled by Bharatiya Janata Party (BJP) governments or, as in the case of Mizoram, by
constituents of the North East Democratic Alliance (NEDA), a BJP-led political platform of anti-Congress parties,
there were expectations that border issues would get resolved. Instead, the situation appears to have become worse.
Monday must serve as a wake-up call for the Centre as well as states in the region.

1. According to the passage which of the following options is true?


(a) In Mizoram, the government is of Indian National Congress.
(b) CMs of both the states had asked Amit Shah to intervene in the dispute.
(c) Recently Assam and Manipur clashed over boundary dispute causing death of Assam police
personnel.
(d) Due to said clash between the two states, 5 police personnel have died who belonged to Manipur.

2. Why did author expect that the border issues would get resolved in the North-eastern region?
(a) Because recently, there has been cooperation and peace in the north-eastern region.
(b) Because the border dispute was resolved in 1972.
(c) Because all the states in the North easter regions are ruled either by BJP or its alliances.
(d) None of the above.

3. Which of the following can be a suitable title for the passage ?


(a) Assam9s encroachment on Mizoram9s land.
(b) North-eastern States of India – A Disturbed Region
(c) Clash between Mizoram and Assam – An Institutional Failure.
(d) Northeast States of India

4. Which of the following means the same as 8simmering9 as it is used in the passage?
(a) carping
(b) tricking
(c) stewing
(d) bickering

Page 2 of 40
5. Which of the following CANNOT be used as a replacement for 8culmination9 as it is used in the passage?
(a) nadir
(b) pinnacle
(c) apogee
(d) apotheosis

Passage (Q.6-Q.10): It has been eight working days since the monsoon session of Parliament began. It started
out with the hope that both the treasury and Opposition benches have a common interest in a functional session. The
government wanted to push through its legislative agenda and was prepared to address the issue of the management
of the second wave of Covid-19 this summer. The Opposition saw an opportunity to hold the government
accountable on public health, price rise and national security. But with the Pegasus revelations storming the Indian
public sphere, the script changed. The Opposition has now demanded that unless there is a full discussion in both
Houses on Pegasus and an enquiry, it won9t allow Parliament to run. The government has issued two perfunctory
statements on the issue but doesn9t want a discussion. And so there is a stalemate.
For the sake of India9s parliamentary democracy, it is crucial to break the stalemate. In principle, the Opposition is
right. The fact that the numbers of key political figures, business executives, bureaucrats, activists and journalists
were on a potential surveillance list, and some of their phone devices were found to be infected with Pegasus 4 a
military-grade spyware 4 is cause for concern. It throws up questions about who was culpable, what it means for
the right to privacy, liberty, dignity, and indeed, the very idea of democracy. Parliament is the best forum to discuss
the issue 4 and it is indeed beyond the purview of just the information technology minister to address it.
The government is in a bind. It cannot admit to procuring and using Pegasus 4 given that hacking is illegal in India
and it will raise difficult questions about its democratic credentials. It cannot deny using Pegasus 4 given that this
means a foreign government was possibly deploying it against Indian citizens. And so it has relied on a defence
based on how all interception and surveillance in India has to be legally sanctioned. But the fact is that at some
point, if not in the House, then in the courts where a bunch of petitions have been lined up, the government will
have to give a detailed response. The government may think that the disruption is costing it little; it is still able to
get legislative business through amid the din and is able to evade more uncomfortable electoral issues such as price
rise. But the spirit of democracy dictates that the Opposition must be allowed its say, even if the government has its
way.

6. According to the passage, why is Government <in a bind=?


(a) It cannot admit that it was using Pegasus because hacking is illegal in India.
(b) It cannot deny using Pegasus because it would mean foreign government was using against Indian.
(c) Both a) and b)
(d) None of the above.

7. Which of the following can be s suitable tone of the passage?


(a) Descriptive
(b) Narrative
(c) Analytical
(d) None of the above.

8. According to the passage, which of the following is not true?


(a) Pegasus is a military grade software.
(b) A number of political figures, business executives, bureaucrats, activists and journalists were on a potential
surveillance list of Pegasus.
(c) It has been eight working days since the Winter session of Parliament began
(d) The Opposition has now demanded that unless there is a full discussion in both Houses on Pegasus.
9. Which of the following DOES NOT mean the same as 8perfunctory9 as it is used in the passage?
(a) desultory
(b) peremptory
(c) cursory
(d) uninterested

10. Which of the following means the same as 8din9 as it is used in the passage?
(a) commotion
(b) business
(c) session
(d) None of the above.

Passage (Q.11-Q.15): When BS Yediyurappa resigned as Karnataka9s chief minister (CM) on Monday morning,
it truly marked the end of an era for both the Bharatiya Janata Party (BJP) and the state. For the party, Mr
Yediyurappa was the last man standing from the old guard who had retained a position of power in the formal
governance structure. Now 78, he joined the Rashtriya Swayamsevak Sangh over five decades ago and rose up the
Jana Sangh and the BJP hierarchy 4 all the way from a district to state unit president; from a member of the
legislative assembly first elected way back in 1983 to a member of Parliament elected in 2014 (the short national
stint was only because he wanted to stay focused on the state); from a three-term leader of opposition in the
assembly to a four-term chief minister, though two of those terms were of less than a week9s duration. But beyond
the formal positions, it was Mr Yediyurappa9s blood, sweat, and tears that enabled the BJP to cross the rubicon
and become a truly national party with an imprint in the south.
In the state, Mr Yediyurappa draws his power from the Lingayat community, which constitutes over 15% of the
population and with a network of sub-religious institutions, wields disproportionate influence over politics. But the
Karnataka satrap also went beyond his caste base to expand the BJP9s footprint among other communities. This
multi-caste alliance has been a strength, but has also generated tensions as is visible in the battle for succession
between leaders of Lingayat and non-Lingayat communities in the state. Mr Yediyurappa9s administrative record is
hard to judge 4 for he never fulfilled a full term in office. But perhaps in a symbol of the close nexus that begun to
mark the nature of India9s compromised crony capitalism in the 2000s, he faced corruption allegations on land and
mining issues, most infamously due to his perceived links with the mining barons of Bellary. This eroded the
party9s image as one with a difference.
The BJP now has to ensure a smooth leadership transition, keeping all factions and castes happy. It also has to
ensure that the exit doesn9t have implications for the stability of the government. Memories of its electoral setback
when Mr Yediyurappa briefly left the party between 2012 and 2014 will be fresh in the BJP9s mind. And it has to
ensure that Mr Yediyurappa does not cash a long shadow over his successor. How it navigates these challenges will
determine the future of the party in the state.

11. According to the passage, which of the following is NOT TRUE about YS Yediyurappa?
(a) He got first elected to Legislative Assembly in 1982.
(b) Mr Yediyurappa draws his power from the Lingayat community, which constitutes over 15% of the
population.
(c) He recently resigned as the CM of Karnataka.
(d) None of the above.

12. According to the author, why is YS Yediyurappa9s political career 8difficult to judge=?
(a) He had lost a huge number of elections before winning one.
(b) He was hated by some people and loved by others.
(c) He never completed a term in the office.
(d) None of the above.

Page 4 of 40
13. What was YS Yediyurappa biggest contribution towards BJP?
(a) He helped BJP reach South India.
(b) He helped vote bank of BJP by including Muslim communities in it.
(c) Both a) and b)
(d) Neither a) nor b)

14. It is written in the passage - it was Mr Yediyurappa9s blood, sweat, and tears that enabled the BJP to cross the
rubicon and become a truly national party with an imprint in the south. What is the meaning of the phrase <cross
the rubicon=?
(a) To become significant in your sphere of work.
(b) To reach a point where you cannot change a decision or course of action.
(c) To be most powerful in a region.
(d) To be able control everything.

15. Which of the following means the same as 8crony9 as it is used in the passage?
(a) dubious
(b) intimate
(c) sacrilegious
(d) vexatious

Passage (Q.16-Q.20): A month ago, both the Bharatiya Janata Party (BJP) and the Congress were struggling
with the political situation in two states where they were the governing party. In Uttar Pradesh, the management of
the second wave triggered criticism of chief minister (CM) Yogi Adityanath; there was also criticism of his
governance style and alleged patronage to certain castes. The BJP national leadership let the controversy play out,
uncharacteristically, in public view. But eventually, it backed Mr Adityanath9s continuation as CM while making
some adjustments, both at the central and state level, to accommodate leaders of other castes. With this, it hopes to
tap into both the incumbency vote (in favour of the CM) as well as the vote of the discontented (who may be
disenchanted with the CM but have faith in the prime minister).
In Punjab, the acrimonious battle between CM Captain Amarinder Singh and rebel Navjot Singh Sidhu played
out, with the national leadership allowing Mr Sidhu to undermine the CM. Eventually, a power- sharing formula
was imposed with Mr Sidhu appointed as party chief. With this, the Congress too hopes to tap into both the pro-
incumbency vote (of supporters of Captain Singh) and the anti-incumbency vote (with Mr Sidhu doubling up both
as party chief but also chief dissenter against the state government). While the context is different, the BJP9s
formula is cleaner to execute 4 back the CM, while ensuring the PM is seen as above the fray. The Congress9s
formula is harder to execute 4 since the CM has to deal with an official rival in the same party structure.
But both national parties have turned their focus now to two other states. For the BJP, it appears a transition in
Karnataka is imminent, with BS Yediyurappa being asked to make way for another leader as CM. The veteran
Karnataka leader has an autonomous base, especially within the Lingayat community, and cannot be brushed aside
easily. How his possible exit impacts the party9s fortunes in the south is to be seen. For the Congress, the next
challenge is Rajasthan, where Sachin Pilot has been waiting for promises to be implemented. The party leadership is
now seeking to ensure the accommodating of his loyalists in the state power structure, but whether Mr Pilot9s rival,
CM Ashok Gehlot, will allow this remains unclear. From UP to Punjab, Karnataka to Rajasthan, both national
parties are attempting to manage state units with an eye on the next election and generational transition, while
ensuring Delhi9s writ runs.

16. According to the passage, which of the following options is NOT TRUE regarding the situation in Punjab?
(a) The governing party in Punjab is BJP.
(b) CM of Punjab is Navjot Singh Sidhu.
(c) Mr Amarinder Singh has been appointed as the party chief.
(d) None of the above.
17. According to the passage, why did BJP include members of certain castes at Centre and State level?
(a) To accumulate the votes of those castes.
(b) Because those members are highly skilled at specific ministerial work
(c) Because the incumbent ministers were not working well.
(d) None of the above

18. Which of the following can be a suitable title for the passage?
(a) Battle between Sidhu and Amarinder Singh in Punjab
(b) Entry of New Ministers in Central Government
(c) Political Adjustments made by BJP and Congress
(d) None of the above.

19. Which of the following can be used as a replacement for 8acrimonious9 as it iss used in the passage?
(a) pectoral
(b) lowly
(c) modicum
(d) rancorous

20. Which of the following does NOT MEAN the same as 8imminent9 as it is used in the passage?
(a) miscible
(b) approaching
(c) impending
(d) forthcoming

Passage (Q.21-Q.25): Mirabai Chanu9s silver medal at the Tokyo Olympics was not unexpected, but it was a
joyous moment when it happened nonetheless. Chanu, who comes from a farming village near Imphal, has been one
of the world9s best weightlifters in her category, 49kg, for some time now. She has won a world championship and
holds the world record in one of the Olympics lifts, the clean & jerk. The Olympic silver simply adds to the aura and
signals, yet again, the pioneering path forged by athletes from Manipur.
Kunjarani Devi, the woman who inspired Chanu, was India9s first medallist at the world weightlifting
championship. She won a silver in 1989, at a time when being a woman weightlifter in India was almost unheard of.
Devi inspired, and continues to inspire, generations of aspiring athletes to take up the sport in India. MC Mary Kom
did the same with boxing 4 opening the way for women to get into a fighting sport and dream of big medals. Six
world titles and an Olympic medal later, she is still fighting, in Tokyo. From the late Dingko Singh to Sarita Devi,
and an assembly line of hockey and football players, Manipur has an outsized influence on the Indian sporting
scene.
Much of that has to do with the love for sport and the sporting culture of the state. Yet, Manipur9s sporting
infrastructure is below par, even compared to the usual less-than-ideal standards everywhere in India. Some states
have, in the past decade, started to tap into sporting culture and encourage its development. Haryana has done that
with boxing and wrestling at the grassroots. Odisha has done so with hockey and athletics and built world class
infrastructure. Karnataka has some of the finest sporting institutes in India now. Manipur would do well to embrace
its unique sporting identity and nurture it in a truly ambitious manner.

21. Which of the following can be a suitable title for the passage?
(a) Enhancing Manipur9s Sports Infrastructure
(b) Mira Bai Chanu and her Achievements
(c) Weightlifting in Olympics
(d) Increase of Women in Olympic Games

Page 6 of 40
22. Which of the following sports athletes in not mentioned in the passage?
(a) Mirabai Chanu
(b) Sarita Devi
(c) Saina Nehwal
(d) MC Mary Kom

23. Which of the following NOT TRUE according to the passage?


(a) Kunjarani Devi, was India9s first medallist at the world weightlifting championship
(b) Mirabai Chanu won a silver medal at the Tokyo Olympics
(c) Mirabai Chanu has won six world titles and an Olympic medal.
(d) Karnataka has some of the finest sporting institutes in India now.

24. Which of the following means the same as 8pioneering9 as it is used in the passage?
(a) titillating
(b) new
(c) scintillating
(d) victorious

25. Which of the following means the closest to 8ambitious9 as it is used in the passage?
(a) blatant
(b) forcible
(c) zealot
(d) determined

Passage (Q.26-Q.30): Mahabaleshwar, a hill station in Maharashtra9s Satara district in the Western Ghats, on
Friday, recorded the highest ever rainfall in its history: 60cm in the previous 24 hours. More than 35 people have
died in rain-induced landslides in the state9s coastal Raigad district. Mumbai continues to be battered by heavy
rainfall and has been placed under orange alert (<heavy to very heavy rain at isolated places=). Other parts of the
western coast, including cities and towns in Madhya Pradesh, Karnataka and Goa, have also recorded exceptional
rainfall and flooding in the last 24 hours. The India Meteorological Department (IMD) classifies rainfall as
<extremely heavy= if an area records 20 cm or more in 24 hours. An offshore trough running from the Maharashtra
coast to north Kerala coast is causing such heavy rains and is likely to weaken from July 26, said IMD.
Over the past month, India has seen several rain-related extreme weather events. Last week, 31 people died in a
series of house collapses after a short burst of intense rain triggered landslides in Mumbai. Both Uttarakhand and
Himachal Pradesh have seen incessant rainfall, with a flash flood in the latter. Delhi has seen some shorter, intense
bursts of rain in the last few days, but only after a slow and delayed start to the monsoon. These wayward weather
events carry not only the indelible sign of a surging climate crisis but are also a compelling reminder that the
country is under-equipped to tackle many of its effects, such as urban flooding and flash floods. But just blaming
the climate crisis will be wrong. Despite some proactive efforts by some states, India9s development plans are
marked by frenzied and unregulated construction, even on flood plains, an utter disregard for natural topography
and hydro-geomorphology, and poor-quality affordable housing. Along with overburdened drainage, these are only
exacerbating the effects of the climate crisis, leading to large-scale human tragedies and destruction of property.
In 100 days, the world will witness the Cop26 summit, the vital United Nations climate talks that open on
November 1 in Glasgow, United Kingdom. The extreme weather events in India and across the world (floods across
Europe and China, wildfires in the United States, killer heatwaves stretching into northern latitudes) are a sharp
reminder of what is at stake, and the express need to accelerate worldwide measures that are required to control the
climate crisis.
26. According to the passage which of the following options is NOT TRUE ?
(a) Mahabaleshwar, a hill station in Maharashtra9s Satara district in the Western Ghats, on Friday,
recorded the highest ever rainfall in its history
(b) In 100 days, the world will witness the Cop24 summit, the vital United Nations climate talks that open in
Glasgow, United Kingdom
(c) Recently, more than 30 people died in a series of house collapses after a short burst of intense rain triggered
landslides in Mumbai.
(d) None of the above.

27. According to the passage which of the following reasons exacerbate the effects of climate crisis in India?
(a) Overpopulation
(b) Unregulated construction
(c) Deforestation
(d) Wastage of water resources

28. Which of the following is not an 8extreme weather event which9 is mentioned in the passage?
(a) Floods in Europe
(b) Wildfires in USA
(c) Heavy Rainfalls in India
(d) None of the above

29. Which of thew following can be used as a replacement for 8wayward9 as it is used in the passage?
(a) pivotal
(b) awry
(c) refractory
(d) equitable

30. Which of the following does not mean the same as 8frenzied9 as it is used in the passage?
(a) quietude
(b) frenetic
(c) turbulent
(d) manic

Page 8 of 40
SECTION-B : GENERAL KNOWLEDGE/CURRENT AFFAIRS

Directions (Q.31–Q.67): Read the information given below and answer the questions based on it.

Passage (Q.31-Q.34): A museum dedicated to the country's 5,000-year maritime history will be set up in
Gujarat's [1] by the Culture Ministry in collaboration with the [2], officials said on Wednesday.

The two ministries signed a Memorandum of Understanding for 'Cooperation in Development of National Maritime
Heritage Complex (NMHC) at [1], Gujarat' on Wednesday.

According to a statement from the Culture Ministry, the world-class facility will be developed in the vicinity of the
Archaeological Survey of India (ASI) site in [1], located about 80 km away from Ahmedabad.

NMHC would be developed as an international tourist destination where the maritime heritage of India -- from
ancient to modern times -- would be showcased and an edutainment approach using the latest technology would be
adopted to spread awareness about the country's maritime heritage, it said.

The NMHC project has been taken up under the unique and innovative projects category of the Sagarmala
programme under MoSPW. The Government of Gujarat has transferred 375 acres of land in Saragvada village on
lease for 99 years at a token rate to MoSPW.

31. Which of the following will replace [1] in the above passage?
(a) Dholavira
(b) Lothal
(c) Pavagadh
(d) Patan

32. Which of the following recently became the UNESCO9s World heritage Site?
(a) Ahmedabad
(b) Rani ki Vav
(c) Dholavira
(d) Champaner

33. Which of the following is not true about the Indus Valley Civilisation?
(a) It flourished around 2,500 BC, in the western part of South Asia, in contemporary Pakistan and Western
India.
(b) The Indus Valley was home to the largest of the four ancient urban civilizations of Egypt,
Mesopotamia, India and China.
(c) In 1929, John Marshall, Director-General of the ASI, announced the discovery of a new civilisation in the Indus
valley to the world.
(d) In 1920s, the Archaeological Department of India carried out excavations in the Indus valley wherein the ruins
of the two old cities, viz. Mohenjodaro and Harappa were unearthed.

34. Which of the following will replace [2] in the above passage?
(a) Ministry of Home Affairs
(b) Ministry of Education
(c) Ministry of Shipping and Port
(d) Ministry of Environment, Forest and Climate Change
Passage (Q.35-Q.38): In the ongoing Monsoon Session of Parliament that ends this Friday, the government will
introduce the [1] Constitution Amendment Bill in Parliament to clarify <some provisions in the 102nd
Constitutional amendment Bill= to restore the power of the states to identify backward classes.

According to the Constitution of India, Articles 15(4), 15(5) and 16(4) confer power on a state to identify and
declare the list of socially and educationally backward classes. As a matter of practice, separate OBC lists are
drawn up by the Centre and each state concerned.

The amendment was necessitated after the Supreme Court in its Maratha reservation ruling in May upheld the
102nd Constitutional Amendment Act but said the president, based on the recommendations of the National
Commission for Backward Classes (NCBC), would determine which communities would be included on the state
OBC list.

"In the task of identification of SEBCs, the President shall be guided by the Commission set up under Article 338B;
its advice shall also be sought by the state in regard to policies that might be framed by it. If the commission
prepares a report concerning matters of identification, such a report has to be shared with the state government,
which is bound to deal with it, in accordance with provisions of Article 338B. However, the final determination
culminates in the exercise undertaken by the President (i.e. the Central Government, under Article 342A (1), by
reason of Article 367 read with Section 3 (8) (b) General Clauses Act)," the judgment read.

35. Which of the following will replace [1] in the above passage?
(a) 127th
(b) 129th
(c) 131st
(d) 135th

36. Which of the following commissions recommended the Maratha reservation?


(a) Justice PN Bhagwati Committee
(b) Justice BP Mandal Commission
(c) Justice NG Gaikwad Commission
(d) Justice Khanwilkar Commission

37. Consider the following about the provision related to Amendment in the Constitution of India:
I. There is no provision of joint sittings on a Constitution Amending Bill.
II. A Constitution Amendment Bill can only be introduced in Lok Sabha
Which of the above is/are true?
(a) Only I
(b) Only II
(c) Both I and II
(d) Neither I nor II

38. Which of the following Supreme Court Judgement held the rule of 50% limit on reservations?
(a) Keshwanand Bharti v. Union of India
(b) M. Nagaraj & Others vs Union of India
(c) Indra Sawhney vs Union of India
(d) Jarnail Singh vs Lachhmi Narain Gupta

Page 10 of 40
Passage (Q.39-Q.42): Prime Minister Narendra Modi announced Friday the renaming of the Rajiv Gandhi Khel
Ratna Award, the country9s highest sporting honour, after hockey player Major Dhyan Chand, setting off a political
war of words with the Opposition Congress.

The Prime Minister said the <exceptional performance= of the men and women hockey teams in the
Tokyo Olympics will rekindle interest in the game across India.

The Congress welcomed the move, but said the Prime Minister <should not have dragged the name of a celebrated
player like Major Dhyan Chand for his myopic political motives=.

It asked him to rename stadiums across the country, including the Narendra Modi stadium in Ahmedabad, after
sports icons. The party accused the Prime Minister of trying to divert attention from issues like the Pegasus row,
farmer protests and rise in prices.

39. When was the Rajiv Gandhi Khel Ratna Established?


(a) 1985
(b) 1988
(c) 1992
(d) 1996

40. Consider the following:


I. Its first recipient was chess legend Viswanathan Anand.
II. Award comprises of a medallion, a certificate, and a cash prize of ₹25 lakh.
Which of the above is/are true?
(a) Only I
(b) Only II
(c) Both I and II
(d) Neither I nor II

41. Major Dhyan Chand was popularly known as which of the following?
(a) The Hockey Genius
(b) The Hockey Magician
(c) The Hockey Wizard
(d) The Hockey Champion

42. Consider the following About Major Dhyan Chand:


I. He was was part of the Olympic team that won gold medals in 1928, 1932 and 1936.
II. Country9s highest award for lifetime achievement in sports is known as the Dhyan Chand Award.
Which of the above is/are true?
(a) Only I
(b) Only II
(c) Both I and II
(d) Neither I nor II
Passage (Q.43-Q.47): A Parliamentary Standing Committee has recommended that India should renegotiate the
Indus Water Treaty (IWT) with Pakistan in view of the <present day pressing issues such as climate change, global
warming and environmental impact assessment=.

Headed by MP Sanjay Jaiswal, the Parliamentary Standing Committee on Water Resources submitted the 12th
report on 8Flood management in the country including international water treaties in the field of water resources
management with particular reference to treaty/agreement entered into with China, Pakistan and Bhutan9 in both the
Lok Sabha and Rajya Sabha on Thursday.

The Committee observed that although the Indus Water Treaty has stood the test of time, it <was framed on the
basis of knowledge and technology existing at the time of its agreement=, when the perspective of both the nations
at that time was confined to river management and usage of water through the construction of dams, barrages,
canals and hydro-power generation.

In recent times, India has often threatened to stop water from flowing into Pakistan, especially in the aftermath of
terror attacks. But the IWT has stood the test of time and has continued to work despite the two neighbours
engaging in three major wars and numerous skirmishes.

43. Indus Water Treaty was signed in which of the following years?
(a) 1956
(b) 1960
(c) 1968
(d) 1970

44. The Treaty under which the commission is formed is backed by which of the
following?
(a) World Bank
(b) United Nations
(c) European Union
(d) USA

45. The Treaty which distributes the water of Indus and its tributaries was signed by
Indian Prime Minister Nehru and Pakistan9s President
(a) Yahya Khan
(b) Zulfikar Ali Bhutto
(c) Iskandar Mirza
(d) Ayyub Khan

46. Consider the following about Parliamentary Committees:


I. Parliamentary committees draw their authority from Article 105 and Article 118.
II. In 1993, 17 Departmentally-related Standing Committees (DRSCs), later increased to 24, were constituted
in the Parliament.
Which of the above is/are true?
(a) Only I
(b) Only II
(c) Both I and II
(d) Neither I nor II

Page 12 of 40
47. In independent India, the first Public Accounts Committee was constituted in which of the following years?
(a) 1950
(b) 1951
(c) 1952
(d) 1954

Passage (Q.48-Q.52): The UN General Assembly approved a resolution Monday establishing a Permanent
Forum of People of African Descent to provide expert advice on addressing the challenges of racism, racial
discrimination, xenophobia and intolerance.

The resolution adopted by consensus by the [1]-member world body also calls for the forum to serve as
<a platform for improving the safety and quality of life and livelihoods of people of African descent= and
their full inclusion in the societies where they live.

The new body's creation comes ahead of the 20th anniversary of the controversial September [2] UN World
Conference Against Racism in Durban, South Africa, which was dominated by clashes over the Middle East and the
legacy of slavery. The US and Israel walked out during the meeting over a draft resolution that singled out Israel for
criticism and likened Zionism to racism.

That language was dropped in the final documents, which condemned and called for the eradication of the scourges
of racism, racial discrimination, xenophobia and intolerance.

48. Which of the following will replace [1] in the above passage?
(a) 189
(b) 190
(c) 193
(d) 196

49. UN declared the International Decade for People of African Descent as which of the following?
(a) 2011-2020
(b) 2014-2024
(c) 2015-2025
(d) 2021-2030

50. Which of the following is not true about Racism?


(a) Every year, 21st March, is observed as International Day for the Elimination of Racial Discrimination.
(b) Article 15, Article 16 and Article 29 of the Constitution of India prohibit discrimination on grounds of
<race=.
(c) India also ratified the International Convention on the Elimination of All Forms of Racial Discrimination
(ICERD) in 1968.
(d) Racial Discrimination is the specific geographical problem of Africa.

51. Which of the following will replace [2] in the above passage?
(a) 2001
(b) 2002
(c) 2003
(d) 2004
52. Consider the following:
I. The forum will consist of 15 members
II. The resolution calls for the forum's first session to take place in 2022.
Which of the above is/are true?
(a) Only I
(b) Only II
(c) Both I and II
(d) Neither I nor II

Passage (Q.53-Q.57): Taliban insurgents tightened their grip on captured Afghan territory on Tuesday, now
controlling 65 per cent of the country, as US President Joe Biden urged the nation's leaders to fight for their
homeland. [1], capital of the northern province of Baghlan, fell to the Taliban on Tuesday evening, according to
residents who reported Afghan security forces retreating towards the Kelagi desert, home to a large Afghan army
base. [1] became the seventh regional capital to come under the control of the militants in about a week. "Afghan
leaders have to come together," Biden told reporters at the White House, saying the Afghan troops outnumber the
Taliban and must want to fight. "They've got to fight for themselves, fight for their nation." The US president said
he does not regret his decision to withdraw, noting that Washington has spent more than $1 trillion over 20 years
and lost thousands of troops. He said the United States continues to provide significant air support, food, equipment
and salaries to Afghan forces. In Kabul, Afghan President Ashraf Ghani said he was seeking help from regional
militias he has squabbled with for years. He appealed to civilians to defend Afghanistan's "democratic fabric".

53. Which of the following replaces [1] in the above passage?


(a) Pul-e-Khumri
(b) Kunduz
(c) Kabul
(d) Herat

54. US-Taliban Agreement for withdrawal of American troops from Afghanistan took place in which of the
following?
(a) Kabul, Afghanistan
(b) Karachi, Pakistan
(c) Doha, Qatar
(d) Abu Dhabi, UAE

55. Which of the following dams in Afghanistan is known as the 8Afghan-India Friendship Dam9?
(a) Kajaki Dam
(b) Grishk Dam
(c) Salma Dam
(d) Dahla Dam

56. Consider the following statements:


I. Taliban forces have gained control of over 65% of Afghanistan.
II. The Taliban recently assassinated Dawa Khan Menapal, Head of the Afghan government's media
information centre in Kabul city.
Which of the above statements is/are true?
(a) Only I
(b) Only II
(c) Both I and II
(d) Neither I nor II

Page 14 of 40
57. India proposed Afghanistan9s membership in the South Asian Association for Regional
Cooperation (SAARC) in which of the following years?
(a) 2001
(b) 2004
(c) 2005
(d) 2002

Passage (Q.58-Q.62): The closing ceremony in Olympic Stadium was fairly relaxed, and perhaps most
poignantly, it aimed to show the athletes a small taste of ordinary life in Japan --something they haven't been
exposed to due to pandemic restrictions.
It wrapped up more than two weeks of athletic competition and the largest international gathering to take place
during the pandemic. The ceremony celebrated the athletes, the volunteers and the organizers of the postponed
Tokyo Games, which involved about 230,000 people, including more than 41,000 people who traveled from abroad.
"You inspired us with this unifying power of sport. This was even more remarkable given the many challenges you
had to face because of the pandemic," [1], the president of the International Olympic Committee, told the athletes.
"You give the world the most precious of gifts – hope."
And the Japanese organizers have now passed the torch to the next city hosting a Summer Games –[2]. The
ceremony kicked off with a video showing some highlights from the events of the Games, and fireworks lit up
the sky. The scenes 4 across countries and sports 4 celebrated the efforts of all the athletes, not medals in
particular.
Then, flag bearers from each country walked in together, in a parade of colorful flags. The U.S. flag bearer is javelin
thrower Kara Winger who was selected by fellow athletes. Japan's flag bearer was karate gold medalist Ryo
Kiyuna.

58. Which of the following replaces [1] in the above passage?


(a) Thomas Bach
(b) Jacques Rogge
(c) Seiko Hashimoto
(d) Yoshiro Mori

59. What is the total number of medals won by India in the Tokyo Olympics 2020?
(a) 9
(b) 6
(c) 7
(d) 8

60. Who is the first Indian individual to win a Gold medal in the Olympic Games history?
(a) Neeraj Chopra
(b) Abhinav Bindra
(c) Manpreet Singh
(d) Milkha Singh

61. Which of the following replaces [2] in the passage?


(a) Brisbane
(b) Paris
(c) Sydney
(d) Shanghai

62. Who among the following was the flag-bearer for India in the closing ceremony of the Tokyo Olympic Games
2020?
(a) Neeraj Chopra (b) Meerabi Chanu (c) Bajrang Punia (d) Susheel Kumar
Passage (Q.63-Q.67): China launched its long-awaited emissions trading system on Friday, a key tool in its quest
to drive down climate change-causing greenhouse gases and go carbon neutral by [1]. The scheme was launched
with China, the world's biggest carbon emitter, seeking to take a global leadership role on the climate crisis in the
lead up to a crucial UN summit in November.
China has hailed it as laying the foundations for what would become the world's biggest carbon trading market,
forcing thousands of Chinese companies to cut their pollution or face deep economic hits.
The programme was launched just days after the [2] unveiled its detailed plan to achieve carbon neutrality by 2050.
However deep questions remain over the limited scale and effectiveness of China's emission trading scheme,
including the low price placed on pollution.
More broadly, analysts and experts say much more needs to be done if China is to meet its environmental targets,
which includes reaching peak emissions by 2030.
China's economic and energy policies are becoming more aligned with the government's environment goals,
according to Zhang Jianyu, vice-president of Environmental Defense Fund China.

63. Which of the following replaces [1] in the passage?


(a) 2050
(b) 2055
(c) 2060
(d) 2065

64. What is the rank of India in the Global Climate Risk Index 2021?
(a) 5th
(b) 7th
(c) 9th
(d) 11th

65. Which of the following countries has zero carbon footprint as of 2021?
(a) Serbia
(b) Suriname
(c) Finland
(d) Denmark

66. Which of the following replaces [2] in the passage?


(a) Korea
(b) European Union
(c) United States of America
(d) Japan

67. Consider the following:


I. India is positioned as the third largest greenhouse gas emitter but also with among the lowest per capita
emissions
II. India9s Paris agreement target is to reach 40% non-fossils by 2030.
Which of the above is/are true?
(a) Only I
(b) Only II
(c) Both I and II
(d) Neither I nor II

Page 16 of 40
SECTION – C: LEGAL REASONING

Directions (Q.68 – Q.104): Read the comprehensions carefully and answer the questions based on it.

Passage(Q.68-Q.72): The Supreme Court of India in 1996 through the K.R. Lakshmanan judgment has devised a
guiding concept to govern the practice of online gaming/gambling. Games, where chance predominates over skill,
are forbidden, while games, where skill prevails over chance, are permitted. In fact, in comparison to other online
games, Courts have been rather demonstrative in establishing a space for fantasy sports. Both the Punjab & Haryana
and the Bombay High Courts have concluded that the games offered by Dream 11 (Fantasy Cricket, Kabaddi, and
Football, among others) demand skill, knowledge, judgment, and attention and are thus not considered to be coming
under the scope of online gambling.
Wagering or betting involves the transaction of money or any type of property to guess the outcome of a race, game,
or any other unpredictable event. Section 30 of the Indian Contract Act makes agreements by way of wager, void,
and unenforceable. The essentials to wager constitute the performance of the bargain depending upon the
determination of an uncertain event where both the parties have mutual chances of gain and loss and where neither
party has control over the happening of the event one way or the other.
Unlike games of chance, in E-sports the amount of skill and talent involved in mastering the game has led to its
professionalization. Also, in E-sports games, the prize money on winning is pre-declared and does not depend on
the number of people joining the contest as in the case of other online games.
[Extracted from 'Here9s how online gaming and E-sports are taxed in India' by Shivani Jha, published 14 June 2021
on The Financial Express

68. Virat created a new gaming platform where football enthusiasts from all over the world could come together and
make their own football teams by selecting players from different clubs. Players could bet on the outcome of the
match, which was dependent on whose team was objectively better. To make an objectively better team, it was
important to have a fundamental understanding of the strengths and weaknesses of each member of the team. Does
this game fall within the scope of online gambling?
(a) No, because only games where chance predominates over skill are forbidden and not games where skill prevails
over chance;
(b) No, because only games where skill prevails over chance are forbidden and not games where chance
predominates over skill;
(c) Yes, because only games where chance predominates over skill are forbidden and not games where skill
prevails over chance;
(d) Yes, because only games where skill prevails over chance are forbidden and not games where chance
predominates over skill.

69. X and Y entered into a contract. The contract stipulated that X would pay 10$ to Y if a head turns up after every two
tosses of a coin and, Y would pay 10$ to X if a tail turns up after every two tosses of a coin. The coin was to be
tossed by Shamsher Bagawat, who was an expert at tossing coins. Based on your reading of the passage, is the
contract between X and Y valid?
(a) Yes, because the performance of the contract depends on the determination of an uncontrollable and uncertain
event where both the parties have mutual chances of gain and loss;
(b) Yes, because the performance of the contract does not depend on the determination of an
uncontrollable and uncertain event as the coin was to be tossed by an expert;
(c) No, because the performance of the contract does not depend on the determination of an
uncontrollable and uncertain event as the coin was to be tossed by an expert;
(d) No, because the performance of the contract depends on the determination of an uncontrollable and uncertain
event where both the parties have mutual chances of gain and loss.
70. Basanti and Gabbar enter into an agreement where Basanti promised to pay Gabbar a sum of Rs. 55,000 if India
wins the world cup. India wins the match and there is an obligation on Basanti to pay the amount to Gabbar but she
fails to do so. Gabbar initiates a suit against Basanti for the recovery Rs. 55,000. Will Gabbar succeed in the
recovery of the agreed amount?
(a) Yes, because the agreement is void and non-enforceable as it is a wagering agreement,
(b) No, because the agreement is based on the loss of one party at the gain of the other,
(c) Yes, because it is dependent on an uncertain event and both the parties have no mutual interest except gain or
loss,
(d) No, because the agreement is void and non-enforceable as it is a wagering agreement.

71. FancyBros Ltd launched a mobile application for a game called Chaar Patti. This game provides a realistic
experience of a casino with real professional dealers, and glamorous studios. Players can play Chaar patti from the
comfort of their homes and pay amounts for each round of cards securely through Paytm. Many novice card players
incurred huge debts on the application while playing Chaar Patti and were unable to pay their debts. Can FancyBros
Ltd recover this amount from these players?
(a) Yes, because playing Chaar Patti involves mutual chances of gain and loss;
(b) Yes, because Chaar Patti is a game of skill and does not constitute online gambling or wagering;
(c) No, because Chaar Patti is a game where skill prevails over chance and not chance predominating over skill;
(d) None of the above

72. Anuradha launched a new company called Laxmi Ltd. The company organised a sweepstake competition where
customers were asked to purchase tickets. A lottery was pulled out and the tickets which contained the number
8539 were awarded cash prizes. Raju had purchased 50 tickets in the hopes of winning a cash prize and ended up
winning Rs. 1 lakh in total. However, when he went to claim the amount, he was turned away. Can he sue Laxmi
Ltd for recovery of the amount?
(a) Yes, because a valid contract was constituted between Raju and Laxmi Ltd when he purchased 50 lottery
tickets;
(b) Yes, because lotteries do not have the characteristics of wagers and are enforceable;
(c) No, because lotteries are fiscally irresponsible and discouraged by the government;
(d) None of the above.

Passage(Q.73-Q.77): In a recent ruling, the Supreme Court of India observed that the 8doctrine of impossibility9
would be equally applicable to Court orders.
The Doctrine of Impossibility stems from the Latin maxim, lex non cogit ad impossibilia, signifying that 8a man
cannot be compelled to do what is impossible9. The genesis of the doctrine is rooted in the law of contracts.
Statutorily enshrined in Section 56 of the Indian Contract Act, 1872, this provision allows contracts to be set aside
due to supervening impossibility preventing its performance.
The application of the doctrine to court orders reflects a shift in the non-performance of a statutory obligation to a
prospective assumption of non-compliance with the Court9s directions by the Executive. Courts often function
within their boundaries to enforce directions for public welfare. Although some of these directions may be stringent,
they go a long way in ensuring that states wake up to the seriousness of the matter concerned. Courts generally take
into account the response of the Executive before issuing directions to them. Thus, it would lead to a lack of clear
demarcation between the mandatory directions issued by the courts and the advice or opinions given in a general
context, which would make way for the Executive to excuse themselves from the directions of the court.
As one of the three pillars of democracy, courts are often approached to ensure that states fulfil their responsibilities
owed to the citizens. This, however, does not excuse the governments for failing their duties on the grounds of
impossibility and impracticality. Such directions cannot be circumvented by claiming 8impossibility9 or
impracticality9 even if it appears to be difficult to implement.
[Extracted with revisions from 'Extending the Doctrine of Impossibility to Court Orders: A Conundrum' by Aisiri
Raj and Vishnu Mohan Naidu, published on NLUJ Criminal law blog

Page 18 of 40
73. During the peak of COVID-19 pandemic, the Allahabad High Court issued several directions such as providing a
minimum number of ambulances in graded towns, requiring new firms to manufacture vaccines by borrowing the
formula from existing manufacturers, providing oxygenated beds in all state nursing homes and upgradation of state
medical colleges within a very short period. The State government contests that these directions are nearly
impossible to implement and the directions regarding the manufacture of vaccines could also have international
ramifications. Can the doctrine of impossibility be invoked against the directions of the Allahabad High Court?
(a) Yes, because the doctrine of impossibility can be invoked against Court orders when there is
supervening impossibility preventing their performance;
(b) Yes, because though directions are difficult to implement and impractical, they are technically possible;
(c) No, because the government cannot be excused for failing their duties on the grounds of impossibility and
impracticality;
(d) No, because invoking the doctrine of impossibility will lead to a lack of clear demarcation between the
mandatory directions issued by the courts and the advice or opinions given in a general context.

74. In the case of River Ganga Organisation v State, the Court ruled in favour of the NGO, River Ganga Organisation.
The NGO had sought a clean-up of River Ganga, including the ghats and the implementation of mandatory
awareness drives, setting up of toilets etc. which was granted by the Court. Three months later, the State said a lack
of funds prevented them from controlling the rampant environmental pollution and fulfilling the court order. Can
the State invoke the doctrine of impossibility to avoid the performance of its duties?
(a) Yes, because a lack of funds can lead to difficulties in the implementation of environmental policies;
(b) Yes, because a lack of funds constitutes a supervening impossibility which prevents performance of the order;
(c) No, because there is a pre-existing duty on the government to address these problems by prioritising them and
allocating a budget to meet these requirements;
(d) No, because a mere difficulty in implementation due to lack of funds does not make the order
impossible.

75. Aladin and Jasmine enter into a contract. Aladin agrees to discover treasure by magic with the help of his Magic
Carpet and a genie. Jasmine had neither seen nor heard of a Magic Carpet or genie but, she promises to give a Rolls
Royce car to Aladin if he performs his part of the bargain. Jasmine grows impatient and demands performance of
the Contract. Based on the principles mentioned in the passage, can she do so?
(a) No, because discovering treasure by magic with the help of a Magic Carpet and a genie is not
impossible;
(b) No, because Aladin cannot be compelled to do what is impossible;
(c) Yes, because Aladin can be compelled to do what is impossible;
(d) Yes, because discovering treasure by magic with the help of a Magic Carpet and a genie is not
impossible.

76. Manjeet booked a banquet hall for his daughter's wedding. The contract with the owner of the hall stipulated that the
owner would be liable to pay damages to Manjeet if he cancelled the booking without a one-month notice. Two
days before the wedding, an unforeseen short circuit culminated into a large fire which burnt down the hall.
Although he felt bad for the owner of the hall, Manjeet was angry because he would not be able to book a venue on
such short notice. Can he seek damages from the owner of the hall?
(a) Yes, because a short circuit does not amount to a supervening impossibility as the owner should not have been
negligent;
(b) No, because a short circuit amounts to a supervening impossibility and renders the contract void;
(c) Yes, because a short circuit amounts to a supervening impossibility that can set aside the contract;
(d) No, because a short circuit amounts to a supervening impossibility that can set aside the contract.
77. Y leased X9s entire restaurant at a rate higher than usual. Y agreed to pay a higher price expecting huge profits as
the Olympics were going on and a large number of tourists were expected in the city. Subsequently, the government
passed an order banning all tourists in the country due to the rising number of COVID-19 cases. Consequently, no
profit was materialised for Y as expected. Y pleaded impossibility because he did not want to pay the higher rent
any longer. Will Y9s plea be accepted?
(a) Yes, because it is impossible for him to attain huge profits in light of the government order and that was the
entire basis of the contract;
(b) Yes, because it is impossible for Y to perform his end of the bargain as he would suffer unprofitability;
(c) No, because mere unprofitability cannot render the contract to be frustrated;
(d) No, because the terms of the contract are not impossible to perform for Y and they are only
unprofitable.

Passage(Q.78-Q.82): The proviso to Section 43D(5) of Unlawful Activities (Prevention) Act [UAPA] lays down
that a court shall not allow bail if <there are reasonable grounds for believing that the accusation against such
person is prima facie true,= thus leading to the belief that courts have no power to grant bail to a person accused
under Sections 15 to 23.
Section 13 mainly concerns itself with disrupting Sovereignty, whereas Section 15 makes punishable, any terrorist
act and opens with the words <any act with intent to threaten or likely to threaten the unity, integrity, sovereignty of
India, or with intent to strike terror or likely to strike terror in the people..." by using lethal weapons, bombs etc.
The conclusion that the court is not to presume the guilt of the accused, and that the "burden to demonstrate the
prima facie veracity, the allegation must fall upon the prosecution" is absolutely correct. When UAPA uses the
words <prima facie true=, it meant that the court must accept the guiltof the accused persons, even if on broad
probabilities. This could happen only if the prosecution had discharged its initial duty of establishing a prima facie
case.
Why this reasoning is sound can be tested by the following simple explanation. At the stage when the court is
considering bail, the only material available with the court is that which has been brought or collected by the
prosecution. So evidently, the court can make up its mind on that material alone which needless to say should be
sufficient for it to arrive at an opinion that prima facie the case is true. [Extracted with revisions from 8Delhi Riots
case: How the Delhi High Court caught the bull by the horns while confronting Section 43(D)5 of UAPA' by
Anjana Prakash, published 17 June 2021 on bar and bench

78. Sharjeel participated in a protest against the established, democratically elected Government of India. The protest
gained widespread media coverage and large masses gathered in the concerned ground. Eventually, the crowd
became so rowdy that police was ordered to do a lathi charge to clear the space. Sharjeel was charged under Section
15 of UAPA. When his case came for hearing, the state submitted pictures of Sharjeel addressing the crowd along
with videos of his speeches. Has the state discharged its burden under Section 43(D)5 of UAPA?
(a) Yes, because the state has submitted enough material on record for the Court to make up its mind and decide
if the accusations are prima facie true;
(b) No, because the state has submitted enough material on record for the Court to make up its mind and decide if
the accusations are prima facie true;
(c) Yes, because the state has not submitted enough material on record for the Court to make up its mind and
decide if the accusations are prima facie true;
(d) No, because the state has not submitted enough material on record for the Court to make up its mind and decide
if the accusations are prima facie true.

Page 20 of 40
79. Based on the evidence presented by the State against Sharjeel, as enumerated in the previous question, the Court
denied Sharjeel9s application of bail. The Court found the allegations against him to be prima facie true. Sharjeel
objected to this decision because he believes that he has a right to bail. Is the disposal of Sharjeel9s bail application
by the Court, a valid action?
(a) Yes, because jail is the exception and bail is the rule in criminal law;
(b) No, because the Court has the power to grant bail in UAPA cases even if the allegations are prima facie true;
(c) Yes, because the Court only needs to make up its mind that the allegations are prima facie true to disallow
bail application;
(d) No, because there is not enough material for the Court to determine if the allegations against Sharjeel are prima
facie true.

80. Tabrez was arrested on charges of rioting by the police of Koregaon for threatening the sovereignty, integrity and
unity of India, under Section 15 of UAPA. Besides the witness statements of a few people who were not even on the
spot of the alleged rioting, the police did not submit any other evidence to substantiate the charges. Can the Court
grant bail to Tabrez in this case?
(a) Yes, because there are reasonable grounds for believing that the accusations against Tabrez are true;
(b) Yes, because the police did not discharge its burden of proving that the allegations against Tabrez are prima
facie true;
(c) Yes, because the police discharged its burden of proving that the allegations against Tabrez are prima
facie true;
(d) No, because there are no reasonable grounds for believing that the accusations against Tabrez are true.

81. P and Q prepared petrol bombs with empty glass bottles of Corona beer. They were loyal citizens and were happy
with the government9s functioning. However, they were very bored with their miserable, monotonous lives and
wanted to get a 8kick9 out of doing something different. As a result, they decided to throw the petrol bombs in a
public park at 2 am in the night when there were hardly any people in the park, so as to create a scare. Can they be
convicted under the provisions of UAPA?
(a) Yes, because they disrupted sovereignty of the nation under Section 13 of UAPA;
(b) Yes, because they intended to strike terror in the people of the nation under Section 15 of UAPA;
(c) Yes, because they committed an act with the intention to threaten the unity and integrity of the nation under
Section 15 of UAPA;
(d) None of the above.

82. P and Q9s antics were published in the newspapers and they were identified as the culprits behind the petrol bomb
incident after a swift police investigation. The Court was informed that materials used in making the petrol bombs,
similar to the ones that were thrown in the park, were found at P and Q9s house. Can the Court grant bail to P and
Q?
(a) Yes, because there are reasonable grounds for believing the accusations against P and Q are prima facie true;
(b) No, because there are no reasonable grounds for believing the accusations against P and Q are prima facie
true;
(c) No, because there are reasonable grounds for believing the accusations against P and Q are prima facie true on
broad probabilities;
(d) No, because there are no reasonable grounds for believing the accusations against P and Q are prima facie
true even on broad probabilities.
Passage(Q.83-Q.88): The offense of criminal breach of trust, as defined under section 405 of IPC, is similar to
the offense of 8embezzlement9 under the English law. A reading of the section suggests that the gist of the offense
of criminal breach of trust is 8dishonest misappropriation9 or 8conversion to own use9 another9s property, which is
nothing but the offense of criminal misappropriation defined u/s 403.
The essential ingredients of the offense of criminal breach of trust are (1) The accused must be entrusted with the
property or with dominion over it, (2) The person so entrusted must use that property, or; (3) The accused must
dishonestly use or dispose of that property or wilfully suffer any other person to do so in violation, (a) of any
direction of law prescribing the mode in which such trust is to be discharged, or; (b) of any legal contract made
touching the discharge of such trust.
For criminal breach of trust, the accused is entrusted with property or with dominion or control over the property.
As the title to the offense itself suggests, entrustment ofproperty is an essential requirement before any offense
under this section takes place. The language of the section is very wide. The words used are 8in any manner
entrusted with property9. So, it extends to entrustments of all kinds-whether to clerks, servants, business partners or
other persons, provided they are holding a position of trust. The term <entrusted= found in Section 405, IPC
governs not only the words <with the property= immediately following it but also the words <or with any dominion
over the property.=
There are two distinct parts involved in the commission of the offense of criminal breach of trust. The first consists
of the creation of an obligation in relation to the property over which dominion or control is acquired by the
accused. The second is misappropriation or dealing with the property dishonestly and contrary to the terms of the
obligation created. The principal ingredients of Criminal Breach of Trust are thus 8entrustment9 and 8dishonest
misappropriation9.

83. The Crime Department is investigating a major bank fraud case. One of the employees X, who is entrusted with
handling the matter has lost an important file which has information on the whereabouts of the accused. Decide.
(a) X has committed criminal breach of trust as he dishonestly disposed of the property;
(b) X has not committed criminal breach of trust as he was not dishonest in disposing of the property;
(c) X has committed criminal breach for he was bribed;
(d) X9s liability cannot be determined.

84. Please refer to the facts above. An intel is received that there was a phone call between the accused and X before the
file went in missing. Decide.
(a) X has committed criminal breach of trust as he dishonestly disposed of the property;
(b) X has not committed criminal breach of trust as he was not dishonest in disposing of the property;
(c) X has committed criminal breach for he was bribed;
(d) X9s liability cannot be determined.

85. Anil and Bhadu are working at the police department. A phone call was traced at Bhadu9s place late at night. In
addition to a phone call, a man wearing plain clothes was also seen taking money to Bhadu9s place and documents
concerning an important case went missing on the next day. This is viewed as credible evidence of bribery. Decide.
(a) Bhadu has committed criminal breach of trust as he dishonestly disposed of the property;
(b) Bhadu has not committed criminal breach of trust as he was not dishonest in disposing of the property;
(c) The very allegation of bribery is enough to show that Bhadu has committed criminal breach of trust;
(d) Bhadu is a public servant within the meaning of the IPC and thus, he cannot be held liable for criminal breach
of trust.

Page 22 of 40
86. Albert wanted to give some documents to his junior. The documents were entrusted to the junior X over a phone
call. X agreed. After hanging up the phone, Albert asked the peon to deliver the documents to X9s office. The file
never reached X9s office.
(a) The peon is liable for criminal breach of trust as he was entrusted with the documents.
(b) The peon is not liable for criminal breach of trust as he was not entrusted with the documents. He only
delivered it.
(c) The peon is liable for criminal breach of trust and X is vicariously liable.
(d) The peon is liable for criminal breach of trust and Albert is vicariously liable for he was the one who entrusted
the file to the peon.

87. Please refer to the facts above. It turns out that the peon had spilt some water onto the file. Scared, he put the file
out in the sun and lied to X about the superior not giving the file. Decide.
(a) The peon is liable for criminal breach of trust as he misappropriated the property with a dishonest intention.
(b) The peon is not liable for criminal breach of trust as he did not have a dishonest intention.
(c) The peon is liable for criminal breach of trust as he lied to X about his superior and thus has a dishonest
intent.
(d) The mere fact that the peon lied about it, is enough to show the presence of dishonest intention.

88. Anil took a file along with him with the intent of taking it away from the police station. Till this time, there is no
evidence of Anil accepting a bribe.
(a) Anil has committed criminal breach of trust as he dishonestly disposed off the property;
(b) Anil has not committed criminal breach of trust as he was not dishonest in disposing of the property;
(c) Anil has not committed criminal breach of trust for he has not misused the property;
(d) Anil9s liability is subject to further evidence of causing the government wrongful loss which is the
accepted metric of a dishonest intention.

Passage(Q.89-Q.94): The offence of criminal conspiracy is defined under Section 120-A of Chapter V-A of the
Indian Penal Code, 1860. According to Section 120-A, when two or more persons come together and agree to do, or
cause something to be done, which constitutes an illegal act or a legal act carried forward by illegal means, such
persons would be guilty of the commission of the offence of criminal conspiracy. In simpler terms, conspiracy
refers to the meeting of minds for the commission of an offence. However, no such agreement would constitute the
offence of criminal conspiracy, unless and until an act is performed in furtherance of such an agreement. The
explanation attached to Section 120-A makes it clear that it is immaterial whether the illegal act committed in
furtherance of such an agreement, is the focal point of the agreement or, is merely incidental to the performance of
the ultimate goal of the agreement.
Salient features: There must be two or more person involved in the commission of the offence of criminal
conspiracy; there must be an agreement between the parties; such an agreement must be for the commission of an
illegal act or the commission of a legal act by illegal means; When the agreement is for the commission of a legal
act by illegal means, an overt act must have been carried out by the parties in furtherance of the same, and the mere
agreement is not sufficient to establish the commission of the crime.
For the establishment of the offence of criminal conspiracy, the existence of an agreement between the parties is a
sine qua non. This agreement may be express or implied, the important factor is consensus ad idem, i.e., meeting of
minds. The agreement must be read as a whole and the object be ascertained. It is not necessary that more than one
person must always be convicted for the offence of criminal conspiracy, it is sufficient if the court is convinced that
more two or persons were actually involved in the conspiracy. The offender might join the conspiracy from the start
itself or at any time before the completion of the objective of the agreement, irrespective of the time of joining, each
party to the offence would be held equally responsible. A criminal conspiracy is said to persist as long as the parties
to the agreement continue to act in furtherance of the objects of the agreement.
89. A and B decided to rob a house of Mr. D. In the process of doing so, A killed Mr. D9s son. Is there a
criminal conspiracy under Section 120A?
(a) Yes, both A and B are part of the criminal conspiracy.
(b) No, only A is responsible for the criminal conspiracy as there was no agreement between A and B for the
murder but only for the robbery.
(c) There is no criminal conspiracy as the essential ingredients for such an act are prior agreement and involvement
of 2 or more people. .
(d) B has not committed criminal conspiracy.

90. In the above facts, if they blackmailed Mr. D9s guard to help them in the robbery, is it a criminal
conspiracy?
(a) Yes, all three of them are part of the criminal conspiracy under Section 120A.
(b) Only A and B form part of the criminal conspiracy.
(c) This is a case of robbery and not criminal conspiracy.
(d) There is no criminal conspiracy since there was no prior agreement between the guard and A & B.

91. On Monday, Ashfaq and Bittu committed theft in a house. In addition, there were three other persons Paras, Qasim
and Rohit who happened to rob the same house on the same day. All five of them were caught by the police. Is there
a commission of criminal conspiracy?
(a) All five of them formed the part of criminal conspiracy.
(b) Only Ashfaq and Bittu formed the part of criminal conspiracy.
(c) Only Paras, Qasim and Rohit formed the part of criminal conspiracy.
(d) There is no criminal conspiracy.

92. In the above facts, Qasim wanted to kill the house owner9s son due to his old rivalry with him but Paras and Rohit
did not agree to this as they only wanted to rob the house. Qasim killed the owner9s son after taking their all money
and jewellery.
(a) All three of them committed criminal conspiracy.
(b) Only Qasim committed criminal conspiracy.
(c) There is no criminal conspiracy as the prior agreement was only of robbing the house and not murder of the
owner9s son. Therefore, Paras and Rohit are not responsible for the murder committed by Qasim as they had
intention to do so.
(d) This is a case of murder and not criminal conspiracy.

93. Paritosh runs a factory that manufactures incense sticks. A few of his workers let in Wazir and Raman who
robbed the factory.
(a) All of them committed criminal conspiracy as there is meeting of minds in the commission of the crime.
(b) Only Wazir and Raman are responsible for criminal conspiracy.
(c) This is a case of robbery and not criminal conspiracy.
(d) No criminal conspiracy is committed.

94. Mr. Bharath is a student of B.E. in Computer Science He loves his computer very much. He considers his computer
as his close friend and companion. On 1.4.2006, while interacting with his computer, he hacked into the Bank
account of Mr. Javed and was successful in withdrawing money front Mr. Javed9s bank account. He did it to please
his girlfriend.
(a) Mr. Bharath has committed an offence.
(b) Mr Bharath has committed cyber crime.
(c) Mr Bharath has committed offence of criminal conspiracy.
(d) Mr.Bharath has not committed the offence of criminal conspiracy.

Page 24 of 40
Passage(Q.95-Q.100): A journalist is not expected to dramatise an incident and place the subject of the news
report at risk; the Allahabad High Court observed while hearing a case where a reporter allegedly egged on a person
to die by suicide in front of the Uttar Pradesh assembly, promising him airtime.
The deceased person's wife is the complainant in this case.
A journalist is not expected to dramatise a sensational and horrifying incident and make news by putting his actor in
a pitiable condition in danger of death.
The bench also spoke on the role of a journalist "The journalist keeps an eye on the anticipated or sudden events
happening in the society and brings them to the information of all the people through various news media without
any tampering; this is his business."
The role of journalists in exacerbating or propagating a situation thrust into public view was also critiqued in the
aftermath of the passing of actor Sushant Singh Rajput last year, when several mainstream television news channels
put Bollywood actors on a media trial and conducted televised investigations on their own, going to the extent of
pronouncing selected people guilty.
If you know someone – a friend or family member – at risk of suicide, please reach out to them. The Suicide
Prevention India Foundation maintains a list of telephone numbers they can call to speak in confidence. Icall, a
counselling service run by TISS, has maintained a crowd sourced list of therapists across the country. You could
also take them to the nearest hospital.

95. Magan is a very prosperous businessman in the community of Hirania Nagar. One fine day, the news issues an alert
of a possible bombing in the area due to an escaped terrorist named Mangal. However, the news fails to display his
name correctly and instead of 'Mangal', displays 'Magan', which leads to a lot of commotion in Magan's vicinity as
he was well known throughout the town. This led to him being insulted and disgraced in his community, which led
to him getting depressed and committing suicide. The Media thereafter issued an apology when the state of affairs
came to its notice. Would the media be liable for his actions herein?
(a) Yes, as his name was severely besmirched, which led to his suicide.
(b) No, as the community knew that he was not the bomber.
(c) Yes, as he was utterly disgraced, and his reputation was destroyed.
(d) No, as any sane person would not equate a well-known businessman with a terrorist bomber.

96. In the above set of facts, had Magan not committed suicide, would the media house have still been liable for his
state of affairs?
(a) No, as he did not commit suicide or harm himself in any other way and as subsequently apologised to.
(b) Yes, due to the false information being published by them, the media house should be held liable.
(c) No, as after issuing an apology, everybody would have realised the mistake.
(d) None of the above.

97. Aaj Tak Sarva Satark is a very well-known news station in North India, with almost all of the households in that
area relying on its bulletins. Recently, they have ventured into the South Indian region with a view to capturing the
populace therein. Consequently, when they receive a hot tip about one big gangster living on the outskirts of
Kerala, they immediately decide to report it. To their dismay, the person living there was a poor fisherman trying to
make ends meet, and someone had pranked them by sending his particulars. This leads to his house being flooded
by police authorities but to no avail, and the fisherman subsequently goes into a shock caused by the entire trauma.
Is the news station liable to be held guilty for exacerbating and dramatising?
(a) Yes, the news station should be held liable as their reputation led the masses to believe that the person
living there was a terrorist.
(b) No, as the news station was a relatively new one in this vicinity and had not yet garnered any support yet.
(c) Yes, as its support is irrelevant here, they reported false news.
(d) No, as the person living there was not a celebrity, and neither did they pose a danger to his life.
98. The same news channel as above reported the relationship of Dhanveer Kapoor and BeepshikaPadukone as being
an abusive and toxic one, wherein both the people involved preach violence. The people involved, however, did not
make any public statement about their relationship and were only ever seen as having dinner once in Mumbai.
However, after this news telecast, they received a lot of backlash from the public, which led to them being pelted
with eggs and their movies flopping. Would this action of the Media make them liable for the danger to the actors'
life?
(a) Yes, as their news roused the public, which led to their movies flopping and them being assaulted.
(b) No, as their relationship was public knowledge anyway.
(c) Yes, as their statements led to the whirlwind of events in which they were caught.
(d) No, as their statements did not endanger their lives.

99. The same news channel as above receives a request by Dhananjaya Bhat; a celeb turned politician who had
allegedly helped a few terrorists enter India through his private cruise. The celeb now wants to confess about the
same. He contacts the news channel stating his request and is granted the same. However, while on primetime, his
phone and anxiety medication is taken by the crew so that the show isn't interrupted. However, this leads to him
having an anxiety attack, and when that gets out of control, he suffers from a stroke which leads to his demise.
Would this fiasco lead to the news station being implicated in the death of the celeb?
(a) Yes, as the actions of the news station led to his ultimate demise.
(b) No, as the news station crew did not force him to surrender his medication.
(c) Yes, as the news channel crew failed to accommodate his health condition.
(d) None of the above.

100. In the above set of facts, after his interview on the news channel, Dhananjaya gets home but is immediately
surrounded by people all around the house and is pelted with stones and fire-lit bottles. This leads to him getting
lynched eventually and the public rioting against his family as being a threat to the sovereignty of the country. The
media learns about this and immediately tries reaching the parents of the celeb and continue to do so even after
being turned down. Would the media be liable here?
(a) The media would be liable here both for the death of the celeb and troubling his parents.
(b) The media would only be liable for troubling his parents.
(c) The media would only be liable for the death of Dhananjaya.
(d) The media will not be liable for either of the events.

Passage(Q.101-Q.104): The Delhi High Court said that a safe house has to be provided for all types of couples-
inter-faith, inter-caste, those belonging to the LGBTQIA+ community - facing opposition from families. While
hearing a petition filed by a couple in a similar situation, Justice Mukta Gupta said.= Whether it9s inter-caste, inter-
religion, inter-community….LGBTQ also covered=. <SHO Police Station Mayur Vihar Phase 1 to ensure
petitioners taken from office and lodged safely at the safe house….there they would be given necessary protection
from SHO through beat staff and other security staff….= The Court ordered. The Court pointed out that the current
arrangement was only a <stop-gap= measure and that the couple9s families had to be made parties to the case in the
future. <Someday they will have to settle on their own. It9s only a stop-gap….They can live at the safe house for
some time but eventually have to move out….You need to implead the family members….=, Justice Gupta said.
Appearing for the petitioners, Advocate Utkarsh Singh submitted that his clients were adults but their families
assaulted them. The petitioners, one of whom was NGO Dhanak For Humanity, argued that the couple had
approached the DCP concerned, but weren9t provided with a safe house as yet. The petitioner also feared being
tracked by the families and the people known to them. Judicial intervention was sought by the couple on the ground
that they also <have the right to pursue their lives with dignity irrespective of their sexual orientation=. Not
providing them with the safe house was violation of their fundamental right guaranteed under Article 21 of the
Constitution of India, it was argued.
Source name: Bar and Bench

Page 26 of 40
101. A and B are couple aged 19 and 17, residing in More Vihaar are in the state of Dailhi. They were threatened by B9s
family that if she doesn9t leave A, they will create trouble for both of them. The couple took the help of the state
police but the police counselled them and told them to return to their respective homes and try to convince their
parents. In the light of the passage given above, select the most appropriate option.
(a) State police is justified in its action because, the girl is a minor and she must not live with her lover in a live-in
relationship.
(b) The girl is a minor and at this tender age, children are not able to take the right decision of their own, so the
state police authority is justified in its action.
(c) Although the girl is a minor, but having A that is her male partner a major, so police should give a helping
hand to the couple from their families.
(d) Although, the girl is a minor and the male partner is a major and they own a right to get protection from the
police authority regarding the matter.

102. P and Q were boys who were in love with each other and called themselves a couple and never hesitated to express
their love in mild way anywhere, even in public. After disclosing their truth to their families, they never objected
their relationship but had a problem of their open expression of the same. After repeated reminder, they didn9t stop
doing the same and ultimately families of the couple ordered them to get separated, negating which the state police
gave them shelter in the safe house. But frequent expression of their love agitated the police and the authority
ordered them to leave the place with immediate effect. In the light of the paragraph, opt out the most appropriate
option.
(a) Providing safe house to a couple resorting to obscenity is nowhere justified and thus, the police have done right.
(b) Every couple has the right to express love anywhere to any extent because now the same sex relation or
marriage has been legalized by the Hon9ble Supreme Court of India.
(c) Couple must be given the allowance to take shelter in the safe house because mild expression of
love is couple9s right and there is no harm in it.
(d) Couple must be given to take shelter in the safe house because they have been threatened by their families if not
got separated from each other.

103. B and C, both aged 17 were couple who took shelter in the safe house and were living there from 3 months escaping
from their family members and their acquaintances who were against their relation. After repeated counselling and
certain advices of the caretaking police authority, they were adamant not to confront their families in any form. The
police resorted to strict action and forced them to leave the safe house and confront their families and convince them
about the same. In the light of the above passage, opt out the most appropriate option.
(a) Actions of the police of forcing the couple to leave the safe was nowhere justified as taking the shelter
in the safe house is their right and giving protection to the couples is police9s duty.
(b) Police9s action was somewhere justified because a couple can9t live like this whole life, and someday, they
have to face the fact and confront their families to convince them. Thus the actions of the police were justified.
(c) Police owns the right and duty to take a decision for the couple for the betterment of the couple as they were
both minors and it is the duty of the police to make them realize their mistakes to ensure a civil society.
(d) Forcing a couple to do anything is nowhere justified and as, the couples were threatened by their family
members, it was the duty of the police to give them protection.
104. From the above passage, which of the option gives a perfect gist of the passage?
(a) Couples have the right to be protected by the police in the safe house irrespective of their explicit expression of
love anywhere to any extent.
(b) Safe house is meant for providing only a temporary shelter to the couples.
(c) Safe house is meant for all the couples irrespective of their sexual orientation but not for minors living in a
relationship.
(d) Police has the authority over the minor to force them out of the safe house to confront their families regarding
their relationship because minors are generally not capable of taking their lives9 decision decently.

Page 28 of 40
SECTION - D: LOGICAL REASONING

Passage(Q.105-Q.109): One hundred years of the CCP is an occasion for the Chinese to celebrate. Most
importantly, disproving Western negativity, the phenomenal success, the unprecedented growth rate and
eliminating poverty for 800 million people are reasons worthy of celebrations. It is not correct to say that the CCP
has not accepted past mistakes and taken the necessary corrective measures. These have been the bases on which
Deng Xiaoping initiated reforms that made possible the economic miracle that followed. The Chinese model that Xi
applauds is based on an impressive track record and its legitimacy is not limited to the renewal of the popular
mandate as we are used to in liberal democracies, but in projecting a political structure that is free from compulsions
of electoral politics.

The entire effort of the CCP is to remind China and the world of the century of humiliation that it suffered because
of the Western design to perpetuate China9s semi-colonial and semi-feudal order. The century that began with the
Opium Wars denied China its rightful evolution and the supremacy that it enjoyed as the longest civilizational state
in the world. Restoring China to its rightful place is not an ideological question but a nationalistic one, which the
CCP intends to achieve by the acquisition of technology and achieving rough parity by facilitating frontier
technology. China is aware of the wide differences between the Cold War periods and accepts the fact of an
interdependent world. The BRI project is an example of that interdependence. The important point for China is the
full restoration of its sovereignty in which all other considerations become secondary.

Nationalism being the driving force, the CCP accepts mistakes and setbacks, but emphasises the economic and
cultural aspects that have made it secure and powerful. This reference by Xi not to accept any bullying is with
regard to the unequal treaties that the Western powers and Japan inflicted on China in the past. The whole emphasis
is about restoring China to its pre-1840 status. The point is that China9s evolution is very different from the West
and the CCP9s legitimacy is based on the centrality of its capacity to guarantee China9s independence. China
reminds the West of its duplicity as well: When the latter invested so heavily, singularly contributing to China9s
success, it did so without insisting on Beijing9s track record on human rights and democratic values. But these have
now become of paramount importance with the rise of China as a formidable power and rival.
[Excerpt from an article by Subrata Mukherjee, The Indian Express, July 5, 2021:

105. All of the following can be inferred except?


(a) Western countries are also responsible for the economic growth of China.
(b) China9s influence internationally has increased substantially in the last few years.
(c) China has become the superpower of the world.
(d) China knows the present world order and takes steps keeping that in mind.

106. Which of the following conclusions can be reasonably drawn from the passage above?
(a) Nationalism is one of the major force behind legitimisation of CCP9s authority in China.
(b) West has become weaker as compared to China in the present world order.
(c) Xi is the most powerful leader China ever had.
(d) None of the above

107. Which of the following weakens the author9s arguments?


(a) China is trying to increase its influence on other countries of the world.
(b) Even after the phenomenal success and unprecedented growth rate of China, poverty in China still exist.
(c) The threat of Chinese acquisition of Taiwan still exist.
(d) None of the above
108. Which of the following is an assumption the author makes?
(a) Human rights and democratic values infringement still exist in China.
(b) CCP has been reasonably successful in strengthening economic prosperity of China.
(c) Both A) and B)
(d) None of the above

109. If author9s arguments in the passage above are true, which of the following is most likely to be true?
(a) West has always been and always will be most powerful in the world.
(b) West has started seeing a more powerful China as its rival.
(c) Western countries are more powerful as compared to China.
(d) All of the above

Passage(Q.110-Q.113): There9s a new virus in town and it9s not fooling around. You can catch it through face-
to-face contact or digitally – that is, via a human or a bot, thanks to technology. Few of us possess immunity, some
are even willing hosts; and, despite all we9ve learned about it, this virus is proving more cunning and harder to
eradicate than anyone could have expected.

Misinformation isn9t new, of course. Fake news was around even before the invention of the printing press,
although the first large-scale journalistic sham occurred in 1835, when the New York Sun published six articles
announcing the discovery of life on the Moon (specifically, unicorns and bipedal beavers). Consider, too, early
modern witch hunts, or those colonial myths that depicted slaves as a different species; the back-and-forth volleys of
anti-Jewish and anti-German propaganda during the world wars, McCarthyism9s Red Scare, even communism9s
utopian narratives. History teems with deceit.

What9s different today is the speed, scope and scale of misinformation, enabled by technology. Online media has
given voice to previously marginalized groups. The transmission of falsehoods now spans a viral cycle in which AI,
professional trolls and our own content-sharing activities help to proliferate and amplify misleading claims.
Moreover, these new developments have come on the heels of rising inequality, falling civic engagement and
fraying social cohesion – trends that render us more susceptible to demagoguery. Just as alarming, a growing body
of research over the past decade is casting doubt on our ability – even our willingness – to resist misinformation in
the face of corrective evidence.
Extracted with edits from Aeon.co:

110. Which among the following best describes the author9s arguments?
(a) The author believes that technology has brought with itself more misinformation to previously
marginalized groups than ever before.
(b) The author talks about the scale of misinformation in the world of this century and ways to mitigate its negative
effects.
(c) The author explains that misinformation has been around for a long time but there is a difference between
misinformation of the past and today.
(d) The author highlights the alarming degree to which misinformation has spread and how people have lost their
ability to resist misinformation.

111. Which among the following can be inferred from the passage?
(a) Our ability to resist misinformation in spite of corrective evidence may vary from time to time.
(b) The perception that technology has acted as a catalyst for misinformation is not free from errors.
(c) The New York Sun incident was one of the first incidents of misinformation to have surfaced.
(d) An important reason for why misinformation has so much endurance is repetition of fake news.

Page 30 of 40
112. Which one of the following, if true, presents a strong confirmation to the author9s arguments?
(a) Online media has given all the resources of deception to the peddlers of mistruth.
(b) The government has been able to mitigate the effects of misinformation.
(c) A dip in ability to resist misinformation does not render us susceptible to demagoguery.
(d) None of the above statements strengthen the author9s arguments.

113. If the information set out in the passage is true, which among the following must be true?
(a) Misinformation can be spread by technology as well as by humans.
(b) The spread of misinformation cannot be altered by anything or any phenomenon.
(c) Both a and b
(d) Neither a nor b

Passage(Q.114-Q.117): Often misunderstood, too much sitting is not synonymous with too little exercise. In
fact, a person considered highly active might sit far too much throughout their day: picture an individual who lifts
weights in the morning and runs for two hours in the evening, but sits in the car during his commute and at a desk
for seven hours throughout the workday. This individual certainly meets physical activity guidelines but might still
be at risk for detrimental health outcomes due to excessive sedentary behavior. It appears that regardless of physical
activity level, increased time sitting corresponds to an increased risk of mortality.
Though the undeniable association between higher risk for disease and greater time spent sitting paints a bleak
picture, research investigating the manner in which sitting time is accumulated offers hope. In one study, one to two
minute breaks from sitting correlated with smaller waists, less insulin resistance, and lower levels of inflammation.
All of the aforementioned areas are risk factors for detrimental health outcomes including cancer and metabolic
disorders. Together, these reports hint that breaks in sedentary time may become a primary method to protect
against its adverse effects.

Despite the pervasive health risks associated with sitting and documented means to temper them, even workplaces,
where people accumulate most of their sedentary time, have enacted very few changes. On the whole, employers do
not brainstorm ways to lessen the time their employees spend sitting and employees do not clamor for opportunities
to escape the confines of their office chairs. Even hiring replacement workers increase company costs, disrupt
normal workplace flow, and raise stress levels. Considering the substantial blow that ailing employees can deal to
companies, it seems more likely that this lack of workplace changes results from ignorance to company policy
modifications. Then, it is probable that considerable workplace changes could take place following heightened
emphasis on the sedentary behavior9s confounding health risks.

114. Which of the following is most supported by the author9s argument?


(a) Anyone who sits down for long will get cancer
(b) No one should sit down for more than an hour any given day
(c) Excessive sedentary behavior is impervious to regular exercise
(d) Some employer9s care about how much time their employees spend sitting down

115. Which of the following can be inferred from the passage?


(a) The impact of sedentary behavior can be controlled by frequent breaks
(b) The present system of working 8 hours a day needs to change
(c) Sitting down is worse when a person is getting some exercise during the day
(d) Both (a) & (b)
116. Which of the following, if true, would seriously undermine the author9s argument?
(a) Majority of people that sit down in their day without a break are able to produce more output than
those who don9t
(b) Majority of people that sit down in their day without a break are uncomfortable with taking a break in between
(c) Both (a) & (b)
(d) Neither (a) nor (b)

117. The statement <People who have a job that keeps them stationary are likely to die earlier= is:
(a) Probably True (b) Definitely True (c) Probably False (d) Definitely False

Passage(Q.118-Q.121): As the Chinese Communist Party (CCP) turned 100, its supreme leader and China9s
President-for-life Xi Jinping claimed his country had never bullied or subjugated any other nation nor would it ever
do so. But the message that rang across the world was exactly the opposite. The reason is simple. Actions speak
louder than words.

The CCP has much to be proud of and as much to be ashamed of. Millions of Chinese perished in the Cultural
Revolution and the Great Leap Forward and an unknown number bore the brunt of its fury in the Tiananmen Square
protests. Not to forget the millions of Uyghurs currently lodged in detention camps in Xinjiang. On the other hand,
the CCP uplifted millions of Chinese from poverty with its Open Doors Policy or rather, the unnatural marriage of
socialism with market economics 4Socialism with Chinese characteristics. The father of this peculiar formulation,
Deng Xiaoping, had cautioned his comrades to bide their time until the country became rich before flexing muscles
on the international stage. Three decades on, Xi surely believes the time has come. China has been clocking a 9.8%
growth rate on an average since 1979 and was the only country to register growth last year despite Covid. Evidently
the most powerful Chinese leader since Mao, Xi has arrogated to himself immense power by ensuring that the two-
term limit on presidency is removed, his own Xi Jinping Thought incorporated in the Constitution and the party
purged of his opponents.

His reign since 2012 has seen China becoming more assertive, be it in the South China Sea or the borders with
India, arm-twisting countries economically, upping the ante on Taiwan and refusing to be transparent about Covid9s
origins. Xi9s Belt and Road Initiative, promotion of authoritarian capitalism and export of digital technology appear
aimed at reshaping the world with China at the center stage. Drum- beating nationalism at home and itching for
confrontations abroad, whether Xi9s tenure spells instability in the CCP remains to be seen. He is definitely in the
process of achieving that abroad.
Extracted with edits from The New Indian Express:

118. Which among the following, if true, presents a strong confirmation to the author9s arguments?
i. The CCP has committed some war crimes inside their nation in the last three decades.
ii. The CCP has done some phenomenal work in getting millions of people out of poverty.
iii. The CCP last year forcefully interfered with the internal affairs of the nation Hong Kong.
(a) Both I and II (b) Both II and III (c) Both I and III (d) All I, II, and III

119. With which among the following views would the author of the passage be most likely to agree with?
(a) The claims of Xi Jinping after the CCP turned 100 has substance.
(b) The throne of Chinese Emperor Xi Jinping is at risk of being taken over.
(c) The time for China about which Deng Xiaoping talked about has come now.
(d) The Chinese Communist Party has not done anything phenomenal during its tenure.

Page 32 of 40
120. Which of the following is an assumption(s) based on which the author9s arguments depend?
(a) Unusual partnerships can sometimes yield positive results.
(b) An average growth rate of 9.8 % per annum for over three decades is uncommon for nations.
(c) Both (a) and (b) are invalid assumptions
(d) Neither (a) nor (b) are invalid assumptions

121. Which question has not been reasonably answered by the question in the passage?
(a) For what works should the Chinese Communist Party be proud of?
(b) What changes has the CCP gone after Xi Jinping came to power?
(c) What are the threats to the power wielded by Xi Jinping?
(d) All the above

Passage(Q.122-Q.126): Dumbledore: In modern day Cricket, full-out aggression is key. The previous days9
<Gentleman9s Game= is gone. Take Virat Kohli for instance, he is known for his aggression and he has captained
his side to the number one ranking team in the world. At a time when the world is in a rat- race for finishing first,
the urge to give it back or to get under the skin of your opponent is the key. It9s a dog-eat-dog world. It was due to
Virat9s infused aggression that India could win the Australian Test Tour even with Virat missing the last 3 of the 4
test matches.

Merlin: Oh yeah? If that is the case then why did India not win the first test match in which Virat was captaining?
They lost the test match then. See, a calm head is what sails you through dangerous waters. I cite Dhoni and
Ajinkya. They are calm in demeanor but aggressive in tactics. Just because you set up an aggressive field for the
opposing batsman, doesn9t mean that you have a license to have a go at the opposing batsman. Making the correct
changes and taking the right steps matter, aggressive or not. It was no wonder that under Ajinkya, India won 2 out
of those last 3 test matches.

122. What does Dumbledore mean when he says that <It9s a dog-eat-dog world=?
(a) The world of now is not that ethical. Winning is the key and for that aggression can be used.
(b) How people behave with us, we should behave with them. If someone is aggressive, we have to be the same
with them.
(c) Only aggression can be able to fight off aggression, otherwise it will overpower you.
(d) All of the above.

123. Which of the following, if true, is a flaw in Merlin9s argument about the first test match in which Virat was
captaining?
(a) Being aggressive in tactic means the person is aggressive. Even though the demeanor is cool, that person will
still be called aggressive.
(b) It was due to bowlers9 dismal performance that India lost the match and nothing else.
(c) Both (a) and (b)
(d) Neither (a) nor (b)

124. Which of the following is an assumption made by Dumbledore?


(a) Aggression was not a part of old days9 cricket
(b) Nowadays all the Cricket teams are aggressive
(c) Both (a) and (b)
(d) Neither (a) nor (b)

125. Which of the following is used by Dumbledore and Merlin to prove their arguments?
1. Examples 2. Idioms and phrases 3. Analogies
(a) Only 1 and 2 (b) Only 2 and 3 (c) Only 1 and 3 (d) None of the above
126. Consider the following -
1. A marketing executive, to be successful, goes to the extent of contacting a competing firm9s
employee and brings out many insider information of its marketing strategy.
2. The marketing executive wants to be successful. He remains calm and does not lose his head when he realizes
his competition is way ahead of him. He calmly proceeds with the existing strategy.
3. Thor was beaten by Thanos in the Infinity Wars. However, in the Endgame Wars, he goes to battle with Thanos
again with the same strategy. But this time, he was less angry than the first time.
4. Iron-Man understands it will be difficult to beat Thanos, as the latter is much more powerful than him. So
instead of trying to fight him, he steals the infinity stones from Thanos – the source of the latter9s power and
then fights him.
Which of the above is a suitable analogy for Merlin9s and Dumbledore9s arguments?
(a) Dumbledore – 1 and 2; Merlin – 3 and 4
(b) Dumbledore – 1 and 3; Merlin – 2 and 4
(c) Dumbledore – 2; Merlin – 3
(d) Dumbledore – 1; Merlin – 4

Passage(Q.127-Q.131):UP9s population policy for 2021-30 unveiled by CM Yogi Adityanath wants to


incentivize couples to stick to a two-child norm. It9s an idea whose time has long gone. The past decade has
witnessed a sharp fall in India9s total fertility rates, even in UP, India9s most populous state. NFHS-4 data from
2015-16 indicates UP9s TFR fell to 2.7 in 2015-16 from 3.8 ten years prior. Sample Registration System report for
2018 pegged UP9s TFR slightly higher at 2.9 in 2018 but even this was a fall of 23.1% in a decade, bettering the
national TFR decline of 18.5%.

Falling TFRs are massive demographic changes catalyzed by education of girls, economic growth, migration,
falling infant mortality rates and higher institutional births. All happened without drastic population control policies.
By all indications, UP will also hit replacement TFR levels of 2.1 in due course like other states.

Apart from being unnecessary, the approach taken by the UP Law Commission9s draft Population (Control,
Stabilization and Welfare) Bill is also dangerous. It has prescriptions portending a bureaucratic nightmare for
ordinary citizens, especially the poor. It requires devoting energies to identifying state employees and the general
public who qualify for a raft of special incentives. Disincentives barring access to welfare schemes and even the
PDS for larger families, though not retrospective, poses exclusion risks, massive corruption and social
discrimination.

Badly conceived laws which trust bureaucracy with inordinate control over people9s lives are a recipe for disaster.
Ironically, the poor, and especially UP9s rural areas where TFR is 3.0 against 2.1 in cities, will be penalized for
structural deficiencies in schooling, public healthcare, and employment opportunities. The potential for widespread
social disruption can even undermine any political gains Adityanath may expect in the UP assembly polls from the
draft bill9s ample references to polygamy. UP should junk this idea.

Extracted from: The Times of India

127. What is the main conclusion drawn up by the author in the passage?
(a) The Uttar Pradesh government should junk its population policy for 2021-30.
(b) Incentivizing couples for having lesser children is a better population control measure than penalizing them for
having more children.
(c) The population policy of Uttar Pradesh government will not serve any meaningful purpose.
(d) Evidence suggests that women in India prefer having lesser children, if given a choice.

Page 34 of 40
128. Which among the following, if true, may weaken the author9s main argument in the passage above?
(a) UP9s population is already coming under control so the population policy of 2021-30 will not have any effect.
(b) Reliable data suggests that India is not being threatened by a 8population explosion9.
(c) Both a and b will not weaken the author9s main argument.
(d) Both a and b will weaken the author9s main argument.

129. Which among the following is an assumption made by the Uttar Pradesh government?
i. Penalizing couples for having more than two children will encourage them to have more children.
ii. Incentivizing couples for having less than two children will positively impact the state9s population
numbers.
iii. The population control measure will ensure that the state9s population of a particular community
declines.
(a) Both I and II (b) Both II and III
(c) All I, II, and III (d) None of the above

130. Which among the following is most likely to be inferred from the passage?
(a) Two-child norm might have been a reasonable measure for population control in the past.
(b) The poor population of Uttar Pradesh will be made to suffer for their past mistakes.
(c) Population control measures that may be dangerous are the way to go in the 21st century.
(d) Results of educating married women about population control have not been encouraging.

131. Which among the following has been used as a premise by the author to support his claims?
(a) The TFR of Uttar Pradesh was 3.8 in 2005-06.
(b) The TFR of Uttar Pradesh is projected to be at 1.7 in 2023-24.
(c) Retrospective effects of the population control measure may trigger massive social discrimination and
corruption.
(d) Both a and c

Passage(Q.132-Q.134): The Indian Medical Association (IMA) recently warned that a third wave of the
pandemic was inevitable and imminent and appealed to the Central and State governments not to let down their
guard against COVID-19. It urged them to not allow mass gatherings in tourist and religious places without
following the COVID-19 appropriate behavior.

In a letter to the Prime Minister, the IMA said that while tourist bonanza, pilgrimage travel and religious fervor all
were needed, they could wait for a few more months. According to it, opening up rituals and enabling people
without vaccination to go scot-free in such mass gatherings were potential super- spreaders of a third wave.

<The past experience of last one and a half years of war with the virus and based on the emerging evidences, it is
obvious that by making the universal vaccination reach the maximum possible population and strictly adopting to
Covid appropriate behaviors, we can face the third wave with confidence and mitigate its impact,99 it stated. The
IMA national president said, <It is the duty and responsibility of everyone at this moment to strictly enforce the
Covid appropriate behaviors for a minimum three more months and ensure everyone gets vaccinated99.

Extracted with edits and revisions from The Hindu


132. Which among the following can be concluded to be true according to the passage?
(a) People who have been vaccinated would not contribute to the possibility of the third wave.
(b) The author agrees with the concerns of the Prime Minister regarding the onset of the third wave.
(c) The author believes that everyone should fulfill their duty of following Covid appropriate behavior at this
moment.
(d) None of the above

133. Based only on the information set out in the passage, with which among the following may be true?
i. Being complacent in following Covid-19 guidelines may invite the third wave.
ii. Pilgrimage travel and religious fervor should be postponed by a few months.
iii. The IMA is concerned about the possibility of a third wave.
(a) Only I and III (b) Only II and III (c) Only III (d) All I, II, and III

134. It is a mistake to see the electoral success of religious majoritarianism in recent years as constructed in a vacuum.
Instead, it is precisely the decades-long compartmentalization of different religious communities, and the absence of
solid state and civil society arrangements in educational pedagogy, personal relationships, workplace, to facilitate
inter-cultural interaction, and based on equality and respect, even under supposedly secular regimes, that has made
the soil politically fertile for the demonization of the minority, especially the Muslims.
Which among the following best summarizes the main idea of the text?
(a) Perceiving that the electoral success of religious majoritarianism was constructed in a vacuum is
inaccurate.
(b) The demonization of the minority including Muslims should be stopped by analyzing the success of the
majority.
(c) Facilitating intercultural interaction presents one of the strongest reasons for the demonization of the minority.
(d) Had there been the presence of civil society arrangements in various spheres of life, the minority would
not be demonized.

Page 36 of 40
SECTION - E :QUANTITATIVE TECHNIQUES

Directions (Q.135 – Q.139): Refer to the table given below and answer the given questions.

Data related to the number of employees in 5 different companies in December 2020

Out of total number of employees


Percentage
Percentage of
Total number of of
Company Percentage of Science graduates commerce
employees Arts
graduates
graduates
X - 30% 30% -
Y - - 40% 20%
Z - 35% 50% -
K 1000 32% - -
L 600 - 42% 30%

Note: Some values are missing, you have find out these value according to the question.
Note: Suppose that all the employees are graduated.

135. What is the difference between the number of commerce graduates employees and Arts graduates employees in
company L?
(a) 12 (b) 18 (c) 10 (d) 22

136. The average number of commerce graduates employees and science graduate employees in company Z was 338.
What was the total number of employees in company Z ?
(a) 1020 (b) 1140 (c) 1040 (d) 1240

137. If the respective ratio between the number of science graduate and commerce graduate employees in company K
was 10 : 7. What was the number of commerce graduate employees in K?
(a) 180 (b) 280 (c) 380 (d) 80

138. Total number of employees in company L increased by 20% from December 2020 to December 2021. If 20% of the
total number of employees in company L in December 2021 was Arts graduate, what was the number of Arts
graduate employees in company L in December 2021?
(a) 144 (b) 169 (c) 244 (d) 104

139. Total number of employees in company X was three time the total number of employees in company Y. If the
difference between number of commerce graduate employees in company Y and that of science graduate employees
in same company was 120, what was the total number of employees in company X?
(a) 600d (b) 1200 (c) 1800 (d) 3000
Directions(Q.140 – Q.144): In Organization A, Ratio of female and male is 6:7. Female at Organization C is
750 more than Male at Organization A. In Organization B, Male and Female are equal. Male at D is 250 more than
the double of Female at A. Female at D is 2550.Male and Female ratio at D is 11:15. Total Male are 5655. And
Male at C is double the Female at A.

140. 20% Female of D are doing MBA and 25% Female of B are doing MBA. What is total no. of
Female doing MBA from these organizations?
(a) 655 (b) 743 (c) 815 (d) 735

141. What is the average (approx.) Male in all the organizations together?
(a) 1414 (b) 5655 (c) 5234 (d) 4567

142. Out of the Total people doing job at A in three departments, marketing, accounts, and IT are in the ratio
of 4:3:2 respectively. Then total people in IT department?
(a) 234 (b) 450 (c) 760 (d) 390

143. Age of some people of every organization is below 30 and some are above 30. 12.5% of total employee from B is
above 30 and25% of total employee of D is above 30. Then find the ratio of above 30 employee of D to the same of
B?
(a) 123:67 (b) 221:61 (c) 223:61 (d) 234:67

144. 20% out of total Female and 25 % out of total Male at C has resigned. What is total number of people who are still
working at C?
(a) 2571 (b) 3456 (c) 3456 (d) 453

Page 38 of 40
Directions(Q.145 – Q.147): Answer the questions based on the information given below.

Out of 200 people who attended a marriage party 100 had GolGape, 120 had Chicken Tica and 40 had none of these
two. 100 people had exactly one of two items.

145. How many people like had both of the given items?
(a) 35 (b) 50 (c) 55 (d) 60

146. How many person does not like Chiken Tica?


(a) 55 (b) 80 (c) 35 (d) 50

147. Person who like only Golgape is what % more or less than the person who like only chicken Tica?
(a) 38.5% (b) 25.6% (c) 15.2% (d) 33.33%

Directions(Q.148 – Q.150): Read the passage carefully and answer the following questions.
X Females can do a work in 2Y days. 1.5X men can do a work in Y days. 8 males, 8 Females and 8 children
together can do the work in 22.5 days. 3X child can do the work in 4Y days.

148. If 2.5X females can complete the work in 24 days. Then Find the value of Y?
(a) 35 (b) 37 (c) 29 (d) 30

149. Find in how many days 10 males and 12 females together can complete the same work.
(a) 19(31/38) (b) 17(31/38) (c) 19(29/38) (d) 17(29/38)

150. How many days 16 females complete the whole work.


(a) 27(5/8) (b) 31(7/38) (c) 31(1/8) (d) 28(1/8)

You might also like